Integration by partial fractions- a quadratic

Click For Summary
To integrate the expression (x-1) / (x^2 - 4x + 5), the denominator cannot be factored into real roots, as it simplifies to (x-2)^2 + 1 upon completing the square. This indicates that the integral should be approached using a different technique rather than traditional partial fraction decomposition. The suggestion is to express the integrand in the form (ax+b)/(Ax^2+Bx+C) due to the absence of real roots. The discussion highlights the importance of recognizing when to complete the square and adjust the integration method accordingly. Understanding these techniques is crucial for solving integrals involving quadratics with complex roots.
TG3
Messages
66
Reaction score
0
Homework Statement

Integrate: (x-1) / (X^2 - 4x +5)

The attempt at a solution

Normally I would try to factor this into something like (x-1) (x+3) (That's an example completely unrelated to this problem.)
However, as no easy factors quickly occurred to me I did a run through of the quadratic equation, and got 4 +or- (16- 4x1x5)^.5 / 2 = 4 + (-4)^.5 / 2

At my basic level of Calculus, imaginary numbers are taboo, so I want to avoid the square root of a negative number.

Am I missing an obvious factoring, am I performing the quadratic wrong (that would be embarrassing) or should I try another technique? And if another technique... what is it?
 
Physics news on Phys.org
well if x^2 - 4x +5=0 has no real roots, then you need to express the partial fraction in the form (ax+b)/(Ax^2+Bx+C)
 
The technique you are missing is to complete the square in the denominator, x^2-4x+5=(x-2)^2+1. Substitute u=x-2. Can you see how to go from there?
 
rock.freak667 said:
well if x^2 - 4x +5=0 has no real roots, then you need to express the partial fraction in the form (ax+b)/(Ax^2+Bx+C)

Quoi? That's the same form as the original integrand!?? It's no partial fraction at all.
 
Question: A clock's minute hand has length 4 and its hour hand has length 3. What is the distance between the tips at the moment when it is increasing most rapidly?(Putnam Exam Question) Answer: Making assumption that both the hands moves at constant angular velocities, the answer is ## \sqrt{7} .## But don't you think this assumption is somewhat doubtful and wrong?

Similar threads

Replies
9
Views
2K
  • · Replies 8 ·
Replies
8
Views
1K
  • · Replies 8 ·
Replies
8
Views
2K
  • · Replies 1 ·
Replies
1
Views
1K
  • · Replies 4 ·
Replies
4
Views
1K
  • · Replies 16 ·
Replies
16
Views
2K
  • · Replies 4 ·
Replies
4
Views
2K
  • · Replies 7 ·
Replies
7
Views
2K
  • · Replies 1 ·
Replies
1
Views
2K
  • · Replies 5 ·
Replies
5
Views
2K